write down the temperature reading(in°c)​

Answers

Answer 1

Answer:

How do you write temperature in Celsius?

Step-by-step explanation:

This is sometimes solved by using the symbol °C (pronounced "degrees Celsius") for a temperature, and C° (pronounced "Celsius degrees") for a temperature interval, although this usage is non-standard.

Hope it helps

Answer 2

Answer:

the answer would be (in words) what ever number is said and then degrees Celsius

Step-by-step explanation:


Related Questions

358.584 divided by 8.92 using mental math and the division algorithm

Answers

Answer:

Below

Step-by-step explanation:

358.584 is about 360      8.92 is about 9

360 / 9 = 40 approx   by' doing it in your head '

ACTUAL answer = 358.584/8.92 = 40.2   Pretty darn close !

You need a $12,000 loan to buy a good used car. Your bank offers a 3-year loan with an APR of 5% and a 5-year loan with an APR of 8%. Find the monthly payment and total interest paid for each loan option. Which would you choose? What is the better deal?

Answers

In case of APR 5% total payment made is $12,947.42 and in case of APR 8% total payment made is $14,598.94. Therefore the better deal to choose to pay the less payment is 1st case.

Define APR.

APR is the annual interest produced by a sum that is paid to investors or charged to borrowers is referred to as the annual percentage rate (APR). This does not account for compounding and includes any fees or additional expenditures related to the transaction. Consumers can evaluate lenders, credit cards, or investment goods using the APR as a benchmark figure.

Initial payment is $12,000 with 5% APR and 3 year loan

Monthly payment = P[ i(1+i)ⁿ] / [(1+i)ⁿ-1]

= 12,000[0.05(1+0.05)³]/[1+0.05)³-1]

=$359.65

Over 36 payments total payment is $12,947.42

And in case of 8% APR with a  5 year loan

Monthly payment = 12,000[0.08(1+0/08)⁵] / [(1+0.08)⁵-1]

=$243.31

Over 60 payments total payment is $14,598.94

So, the best deal is 5% APR with a 3 year loan.

To know more about APR visit:

https://brainly.com/question/14184570

#SPJ1

PLEASE HELP IM TIMED I WILL GIVE BRAINLIST!!!!!!!!

Answers

The distance between the line segments PQ, RS and TV are √53 respectively

Distance Between Two Points

The distance between any two points is the length of the line segment joining the points. There is only one line passing through two points. So, the distance between two points can be calculated by finding the length of this line segment connecting the two points.

The formula is given as;

d = √(x₂ - x₁)² + (y₂ - y₁)²

In the first line segment;

P = (0,4), Q = (7, 2)

d = √((7-0)² + (2 - 4)²)

d = √53

b)

R = (-7, 6), S = (-5, -1)

d = √((-1 - 6)² + (-5 --7)²)

d = √53

c)

T = (0, -8), V = (2, -1)

d = √((-1 --8)² + (2 - 0)²)

d = √53

The distance between the lines are all √53 units

Learn more on distance between two points in a line segment here;

https://brainly.com/question/7243416

#SPJ1

What is the solution and how do I solve?

Answers

20 + 4x+2 = 6x+8 --- By exterior angle

4x+22=6x+8

4x-6x=8-22

-2x=-14

x=7

Hope it helps

20 + 4x+2 = 6x+8
22 + 4x = 6x+8
4x-6x = 8-22
-2x = -14
x=7
Answer: x=7

Can someone do these 4 questions For 40 points

Answers

The gradient of the blue line as shown in the graph is 0.25

What is gradient?

The gradient of any line or curve tells us the rate of change of one variable with respect to another.

To calculate the gradient of the blue line, we use the formula below.

Formula:

S = Δy/Δx = (y₂-y₁)/(x₂-x₁)................. Equation 1

Where:

S = Gradient of the blue lineΔy = Change in y-axisΔx = Change in x-axis

From the graph,

Given:

x₁ = 0y₁ = 1x₂ = 8y₂ = 3

Substitute these values into equation 1

S = (3-1)/(8-0)S = 2/8S = 1/4S = 0.25

Hence, the gradient of the blue line is 0.25.

Learn more about gradient here: https://brainly.com/question/21727173

#SPJ1

Work out the surface area of this solid
prism.
10cm
8cm
27cm
6cm
17cm
30cm
The diagram is not drawn to scale.

Answers

Answer:

  2064 cm²

Step-by-step explanation:

You want the surface area of a prism that is 30 cm between bases that are trapezoids with parallel sides 27 cm and 6 cm that are 8 cm apart, and slant sides that are 10 cm and 17 cm.

Base area

The area of each trapezoidal base is ...

  A = 1/2(b1 +b2)h

There are two identical bases, so their total area is ...

  2A = (b1 +b2)h = (27 cm +6 cm)(8 cm) = 264 cm²

Lateral area

The lateral area of the prism is the area of the four rectangular faces. Each is 30 cm wide, and their total length is the perimeter of the base;

  P = 27 cm +10 cm +6 cm +17 cm = 60 cm

  lateral area = Ph = (60 cm)(30 cm) = 1800 cm²

Total surface area

The total surface area of the prism is the sum of the base area and the lateral area:

  total area = base area + lateral area

  total area = 264 cm² +1800 cm² = 2064 cm²

The surface area of the solid prism is 2064 cm².


Dajon already knows that he will have $500,000 when he retires. If he sets up a payout annuity for 15 years
in an account paying 2.05% interest, how much could the annuity provide each week? Round your answer to
the nearest dollar.

Answers

The annuity provide each week is $547.59 (approx.)

Define Annuity

A fixed sum of money paid to someone each year, typically for the rest of their life.

Given,

Dajon already knows that he will have $500,000 when he retires.

P = $500,000

Next, he sets up a payout annuity for 15 years in an account paying 2.05% interest.

r = 2.05 % or 0.0205

t = 15 years

Since, annuity provide each week then

n = 52 weeks (in a year)

The Annuity formula is,

P = d [ ( 1 + r/n)^nt - 1 ] / (r/n)

where, d = regular deposit

Now, plug in the values and find the value of d

P = d [ ( 1 + 0.0205/52)^52*15 - 1 ] / (0.0205/52)

Calculate the value it gives,

P = $547.5936993 or 547.59

Hence, the annuity provide each week is $547.59 (approx.)

To read more about Annuity

https://brainly.com/question/13722779

#SPJ1

A tortoise and a hare are competing in a 2000-meter race. The arrogant hare decides to let the tortoise have a 820-meter head start. When the start gun is fired the hare begins running at a constant speed of 7.5 meters per second and the tortoise begins crawling at a constant speed of 4.5 meters per second.
a. Write an expression in terms of t that represents the hare's distance from the starting line (in meters)
b. Write an expression in terms of t that represents the tortoise's distance from the starting line (in meters).
c. Write an expression in terms of t that represents the number of meters the tortoise is ahead of the hare.

Answers

a. The hare's distance from the starting line is represented by the expression 7.5t.

b. The tortoise's distance from the starting line is represented by the expression 4.5t+820.

c. The number of meters the tortoise is ahead of the hare is represented by the expression 4.5t+820-7.5t.

In the race, the hare is running at a constant speed of 7.5 meters per second, so their distance from the starting line is equal to their speed multiplied by the time it takes them to run that distance. This is represented by the expression 7.5t, where t is the time in seconds it takes the hare to run.

Similarly, the tortoise is crawling at a constant speed of 4.5 meters per second, so their distance from the starting line is equal to their speed multiplied by the time it takes them to crawl that distance, plus the 820-meter head start they were given. This is represented by the expression 4.5t+820, where t is the time in seconds it takes the tortoise to crawl.

To find the number of meters the tortoise is ahead of the hare, we subtract the hare's distance from the starting line from the tortoise's distance from the starting line, which gives us the expression 4.5t+820-7.5t.

Learn more about Linear Expressions here:

https://brainly.com/question/25682757

#SPJ4

-15x + 20y = 15
0=18x + 24y - 18

Answers

Answer:

Step-by-step explanation:

given:

-15x + 20y =15

0 = 18x + 24y -18

let's get the 2nd equation into a form like the top equation

-18x -24y = -18

multipy thur by -1

18x +24y = 18

we'll need to  use substitution for this problem.  elimination has a zero issue.

18x = -24y +18

x = -[tex]\frac{4}{3}[/tex] y + 1

now plug this into the 1st equation

-15(  -[tex]\frac{4}{3}[/tex] y + 1 ) + 20 y = 15

20y -15 +20y = 15

40y = 30

y = 3/4

now that we've found y, plug that into our x = -[tex]\frac{4}{3}[/tex] y + 1 , to find x

x = -1 +1

x = 0

A cake recipe asks for 1/4 cup of oil for each cake. How many cakes can be made from a bottle of oil that has 4 cups in it?

Answers

Answer: 16

Step-by-step explanation:

there are 4 1/4 in 1 cup. so it'll be 4 cakes for each cup of oil you have. multiply the number of cakes that can be made per cup (4) by the number of cups you have (4)

(4) x (4) = 16

Yes, by ASA and AAS
yes SAS
Yes by HL
yes by SSS
the two triangles are not congruent

Answers

Answer:

yes, by ASA

Step-by-step explanation:

sum of interior angles of a triangle = 180°

180 - (45+63) = 180 - 108 = 72°

the two triangles have two congruent angles and the side between them.

They are congruent for (ASA) angle side angle

The maximum weight, w, that an elevator can hold is 2200 pounds. Which inequality represents the situation?

Answers

Answer: w ≤ 2200 would represent the situation

Which of the following is a true statement? A. All real numbers are rational numbers. B. All whole numbers are natural numbers. C. All integers are whole numbers. D. All natural numbers are integers.​

Answers

Answer: It is D

Step-by-step explanation:

1/4+1/6 in fractions
Please help me I’m giving you 25 pints things

Answers

Write common denominator

3+2/12

Answer: 5/12
5/12

Step by step:
1)Find the LCM of 6 and 4 =12
2)set both fractions to over 12 and do whatever you done to the bottom to the top
3)1x3=3 1x2=2
4)3+2 =5
Answer: 5/12

work out the area of a rectangle with base, b = 9.6mm and hight, h = 2mm

Answers

Answer: 19.2mm^2

Step-by-step explanation: the area of a rectangle is simply base x height

so do 9.6 x 2 to get 19.2mm^2

Using the income statement below, calculate the following profitability ratios for Western Bookkeeping and Tax Service. Assume that stockholder's equity equals $441,600 and total assets equal $640,000.
Profit margin ratio
Return on equity ratio
Asset turnover ratio
Write the profit margin and return on equity ratios as a percent, rounded to the nearest percent. Write the asset turnover ratio as a decimal, rounded to two decimal places.

Answers

The computation of the profitability ratios for Western Bookkeeping and Tax Service is as follows:

Profit margin ratio = 8%Return on equity ratio = 8%Asset turnover ratio = 0.75.

What are profitability ratios?

Profitability ratios are the financial ratios that evaluate an entity's ability to convert its earnings (revenue) into profit versus different criteria.

Some of the common profitability ratios include:

Gross Profit MarginOperating Income RatioNet Income RatioReturn on Investment (ROI)Return on EquityReturn on Total AssetsAsset turnover RatioEarnings per share.

Stockholders' equity = $442,600

Total assets = $640,000

Net sales = $480,000

Net income = $36,000

Profit margin ratio = Net income/Net Sales x 100

= $36,000/$480,000 x 100

= 7.5%

= 8%

Return on equity ratio = Net income/Shareholders' Equity x 100

= $36,000/$442,600 x 100

= 8.13%

= 8%

Asset turnover ratio = Net Sales/Total Assets

= $480,000/$640,000

= 0.75

Learn more about the profitability ratios at https://brainly.com/question/16750058

#SPJ1

One factor of the function f(x) = x3 − 6x2 + 11x − 6 is (x − 3). Describe how to find the x-intercepts and the y-intercept of the graph of f(x) without using technology. Show your work and include all intercepts in your answer

Answers

One factor of the function f(x) = x3 − 6x2 + 11x − 6 is (x − 3). the x-intercepts and the y-intercept of the graph of f(x) are

x = 2, x = 3, and x = 4.(0, -6), (2, 0), (3, 0), and (4, 0). Respectively

What are the x-intercepts?

Generally, To find the x-intercepts, we set y equal to 0 and solve for x. In this case, the equation we need to solve is:

f(x) = x^3 - 6x^2 + 11x - 6 = 0

To solve this equation, we can use the fact that (x - 3) is a factor of the function. This means that we can write the function as:

f(x) = (x - 3)(x^2 - 3x + 2)

If (x - 3) is a factor, then x - 3 must equal 0. This gives us one x-intercept at x = 3.

To find the other x-intercepts, we can set the quadratic factor (x^2 - 3x + 2) equal to 0 and solve for x. This gives us the following equation:

x^2 - 3x + 2 = 0

We can use the quadratic formula to solve this equation:

x = (3 +/- √(9 - 8)) / 2 = (3 +/- √(1)) / 2 = (3 +/- 1) / 2

This gives us two more x-intercepts at x = 2 and x = 4.

So the x-intercepts of the graph of f(x) are x = 2, x = 3, and x = 4.

To find the y-intercept, we set x equal to 0 and solve for y. In this case, the equation we need to solve is:

f(0) = 0^3 - 6(0)^2 + 11(0) - 6 = -6

This gives us a y-intercept at (0, -6).

So the y-intercept of the graph of f(x) is (0, -6).

All of the intercepts of the graph of f(x) are, therefore (0, -6), (2, 0), (3, 0), and (4, 0).

Read more about x-intercepts

https://brainly.com/question/14180189

#SPJ1

Can someone evaluate these for me
1×(– 6 × –18 –8) ÷4
4– – 20+ – 5 ×( –11 – –18)
13– – 10 × 16 ÷ (14 + –16)
1– – 9 × 12 ×(–19 +20)

Answers

The answer of the given expression using BODMAD rule is 25.

What is BODMAS rule?

The BODMAS rule states that the brackets must be solved first, then powers or roots (i.e., of), Division, Multiplication, Addition, and finally Subtraction. Only when the BODMAS rule or the PEDMAS rule is used to solve an expression is the solution deemed to be correct.

(a) 1 x (-6 x -18 - 8) ÷ 4

By using first to simplify the bracket.

1 x ((-6 x -18) - 8) ÷ 4

1 x (108 - 8) ÷ 4

(1 x 100) ÷ 4

100 ÷ 4

25

Hence, the answer of the given expression using BODMAD rule is 25.

To know more about the BODMAS rule, click on the link

https://brainly.com/question/29626868

#SPJ1

Algebraically prove that x^3 + 9 / x^3+8 = 1 + 1/x^3 +8 , where x ≠ -2

Answers

Refer to the photo taken.


Eileen is making lemonade with water and lemon concentrate. What percent of each mixture is
lemon concentrate?
a. The ratio of water to lemon concentrate is 3 to 1.
b. Eileen mixes 4 parts water for each part of lemon concentrate.
c. 4 out of every 5 cups of the mixture is water.

Answers

a) The percentage of lemon is 25%

b) The percentage of lemon is 20%

c) The percentage of lemon is 20%

What is a percentage?

The percentage is calculated by dividing the required value by the total value and multiplying by 100.

Example:

Required percentage value = a

total value = b

Percentage = a/b x 100

Example:

50% = 50/100 = 1/2

25% = 25/100 = 1/4

20% = 20/100 = 1/5

10% = 10/100 = 1/10

We have,

a)

Water : Lemon = 3 : 1

Percent of lemon concentrate:

= 1/4 x 100

= 25%

b)

1 part of lemon = 4 part of water

Percent of lemon.

= 1/5 x 100

= 20%

c)

Cups of lemon = 1

Cups of water = 4

Percent of lemon.

= 1/5 x 100

=  20%

Thus,

The percent of each answer is given above.

Learn more about percentages here:

https://brainly.com/question/11403063

#SPJ1

r²-6 r+9 can be factorized to give an expression of the form (r+a)²,
where a is an integer.
Work out the value of a.

Answers

The value of a in the expression is -3

How to determine the value of a?

From the question, we have the following parameters that can be used in our computation:

r²-6 r+9 can be factorized to give an expression of the form (r+a)²

This means that

r² - 6r + 9 = (r+a)²

Factorize the expression r² - 6r + 9

So, we have

(r - 3)² = (r + a)²

By comparison, we have

a = -3

Hence, the solution of a is -3

Read more about factored expression at

https://brainly.com/question/28720413

#SPJ1

Graph by finding the x and y intercepts using a table

-3x-6y=0

Answers

Answer: see below

Step-by-step explanation:

x=0 -3(0)-6y=0 -6y=0 y=0

x=1 -3(1)-6y=0       -4-6y=0    -6y=4      y=4/6 or 2/3

x=2 -3(2)-6y=0      -6-6y=0      -6y=6    y=-1    

x=3 -3(3)-6y=0         -9-6y=0        -6y=9       y= -9/6   or -3/2

x=-1 -3(-1)-6y=0          3-6y=0           -6y=-3       y=1/2  

x              y

-1               1/2

0              0

1               -2/3

2              -1

3             -1 1/2

A student receives a subsidized student loan for $10,000 with a 10-year repayment term and interest compounded monthly. Six months after she graduates from college, she will have to begin making payments. Find her monthly payment and total interest paid if the interest rate is 5.5%.

Answers

If the interest rate is 5%, the monthly payment and total interest would be $2.439.

Define interest.

Interest is the sum of money that must be repaid on a loan or received in exchange for a loan. Principal refers to the borrowed or lent funds. The Amount is the total of the principal plus interest. The term "rate of interest" refers to the rate at which interest is applied to the principal.

Given,

A student receives a subsidized student loan for $10,000 with a 10-year repayment term and interest compounded monthly. Six months after she graduates from college, she will have to begin making payments.

A $10,000 subsidized student loan

Ten-year term of repayment

5.5% interest rate

First, we must determine the loan's future value (FV) over the next six months using the formula below:

FV in 6 months

= Loan Amount× (1 + Interest Rate)⁶ ($10,000×(1 + 0.055) ×6

= $2.7680.

Second, the formula for determining future value is used to determine the monthly payment:

M = FV in 6 months / (((((r)n) - 1) / r).......... (1)

When you enter all the pertinent values into equation (1), you get:

M = $2.7680 / ((((1 + 0.00466666666666667)^144) – 1) / 0.00466666666666667)

M = $2.439

Therefore, if the interest rate is 5.5%, the total amount paid in interest plus monthly payments would be $2.439.

To learn more about interest, visit:

https://brainly.com/question/28792777

#SPJ1

Nine less than two times a number is five more than this number.

Answers

The statement as an expression is 2x - 9 = 5 + x

How to determine the statement as an expression?

From the question, we have the following statement that can be used in our computation:

Nine less than two times a number is five more than this number.

Express the numbers properly

So, we have the following representation

9 less than 2 times a number is 5 more than this number.

Introduce the equality symbols

So, we have the following representation

9 less than 2 times a number = 5 greater than this number.

Next, we apply the mathematical operators

So, we have the following representation

2 times a number - 9 = 5 + number.

Express the number as x

So, we have

2x - 9 = 5 + x

Hence, the expression is 2x - 9 = 5 + x

Read more about expressions at

https://brainly.com/question/15775046

#SPJ1

What are the solutions to this system? Select all that apply. Y+2x^2 =14
y= 3x^2-6

Answers

The solution to the system of equation y + 2x² = 14 , y = 3x² - 6 is as follows:

x = 2y = 6

How to solve system of equation?

System of equation can be solved using different method such as elimination method, substitution method and graphical method.

Let's solve the system of equation by substitution method.

Therefore,

y + 2x² = 14

y = 3x² - 6

Therefore, using substitution method,

3x² - 6 + 2x² = 14

5x² - 6  = 14

5x² = 14 + 6

5x² = 20

x² = 20 / 5

x² = 4

x = √4

x = 2

Therefore, let's find the value of y using y = 3x² - 6.

y = 3x² - 6

y = 3(2)² - 6

y = 12 - 6

y = 6

learn more on system of equation here: https://brainly.com/question/12895249

#SPJ1

Solve for m in the equation below. It may be helpful to convert the equation into exponential form. Write answer as an integer or reduced fraction. -2. log,(m) - 24 = - 20 m = > Next Question Find the largest value of 2 that satisfies: log, () – log (2 + 1) = 9 =

Answers

The largest possible value of 2 that satisfies the equation is approximately 1.1.

What is Exponential Form Equation?

Since y=bx, y = b x is a one-to-one inverse of the exponential function, x=by x = b y, it too is a function. We simply swap x and y and solve for y to discover the inverse function, as is the case with all inverse functions.

To solve for m in the equation -2 * log(m) - 24 = -20, we can start by converting the equation into exponential form. To do this, we can raise both sides of the equation to the power of e:

[tex]e^(-2 * log(m) - 24) = e^(-20)[/tex]

Then we can simplify:

[tex]m^(-2) = e^(-20)[/tex]

Then we can solve for m:

          [tex]m = sqrt[e^20]\\\\= sqrt[20*e^9]\\\\= sqrt[20] * sqrt[e^9]\\\\= 2 * 3^(1/2)= 2 * 1.732\\\\= 3.464[/tex]

So m is approximately equal to 3.464

To find the largest value of 2 that satisfies the equation [tex]log(2^2) - log(2 + 1) = 9[/tex],  we can start by expanding the left-hand side of the equation:

       [tex]log(2^2) - log(2 + 1) \\\\=log(2^2 / (2 + 1))\\\\ = log(4/3) = 9[/tex]

Then we can rewrite the equation as:

log(4/3) = 9

Then we can solve for 2:

         [tex]2 = (4/3)^(1/9)\\\\= (4^(1/9)) / 3^(1/9)\\\\= 1.5874010519681994 / 1.4422495703074083\\\\= 1.1[/tex]

So, The largest possible value of 2 that satisfies the equation is approximately 1.1.

To know more about Exponential Form Equation visit,

https://brainly.com/question/23275698

#SPJ4

Can someone help me with this question????

Answers

Answer:

[tex]P = \boxed{\sf 4000}[/tex]

[tex]r=\boxed{\sf 0.03}[/tex]

[tex]n=\boxed{\sf 4}[/tex]

[tex]t=\boxed{\sf 5}[/tex]

Step-by-step explanation:

[tex]\boxed{\begin{minipage}{8.5 cm}\underline{Compound Interest Formula}\\\\$ A=P\left(1+\frac{r}{n}\right)^{nt}$\\\\where:\\\\ \phantom{ww}$\bullet$ $A =$ final amount \\ \phantom{ww}$\bullet$ $P =$ principal amount \\ \phantom{ww}$\bullet$ $r =$ interest rate (in decimal form) \\ \phantom{ww}$\bullet$ $n =$ number of times interest is applied per year \\ \phantom{ww}$\bullet$ $t =$ time (in years) \\ \end{minipage}}[/tex]

If you invest $4,000 then the principal amount is $4,000.  As P represents the principal amount:

[tex]\implies P = \boxed{\sf 4000}[/tex]

The interest is 3%.  3% = 3/100 = 0.03.  Therefore:

[tex]\implies r=\boxed{\sf 0.03}[/tex]

"n" represents the number of times interest is applied per year.  

Therefore, if the interest is applied quarterly then:

[tex]\implies n=\boxed{\sf 4}[/tex]

"t" represents the time in years.  Therefore, if you plan to leave the money in the account for 5 years then:

[tex]\implies t=\boxed{\sf 5}[/tex]

-----------------------------------------------------------------------------------------

To calculate the amount in the account after 5 years, substitute the values into the formula and solve for A:

[tex]\implies A=4000\left(1+\dfrac{0.03}{4}\right)^{4 \times 5}[/tex]

[tex]\implies A=4000\left(1+0.0075\right)^{20}[/tex]

[tex]\implies A=4000\left(1.0075\right)^{20}[/tex]

[tex]\implies A=4000(1.16118414...)[/tex]

[tex]\implies A=\$4644.74[/tex]

The art museum is 8 1/2 miles from Alison's house. Alison has ridden her bike 2/3 of the way there so far. How far has she gone?

Answers

Alison has traveled six miles on her bike.

What is the distance?

Distance is defined as the product of speed and time.

To find how far Alison has ridden her bike, we need to first convert 8 1/2 miles to miles as a mixed number.

We can do this by multiplying the fractional part of the mixed number by the denominator of the fraction:

8 1/2 miles = 8 miles + (1/2 x 2) miles = 8 miles + 1 mile = 9 miles

We can now multiply this distance by 2/3 to find how far Alison has ridden her bike:

9 miles x 2/3 = 6 miles

Therefore, she has ridden her bike 6 miles.

Learn more about the fractions here:

brainly.com/question/10354322

#SPJ1

I NEED HELP ASAPPPPPP

Answers

Answer:

2.5

Step-by-step explanation:

m is the gradient

to find this we do the change in y/change in x

using the first 2 values we have been given:

change in y= 15-10

change in x= 4-2

5/2= 2.5

the gradient in 2.5

American children watch an average of 25 hours of television per week with a standard deviation of 8 hours. A random sample of 40 children is selected. What is the probability the mean number of hours of television they watch per week is less than 22? Question 15 options: 1) 0.3520 2) 0.0089 3) 0.9911 4) 0.6480

Answers

If American children watch an average of 25 hours of television per week with a standard deviation of 8 hours. the probability the mean number of hours of television they watch per week is less than 22 is: 2) 0.0089.

How to find the probability?

First step is to find the standard deviation

σ = 8/√40

σ = 1.2649

Using this formula to find the Z-score

Z = x - μ/σ

Z =22-25/1.2649

Z = -3//1.2649

Z = -2.37

Hence,

P-value=P(Z < -2.37)  

P-value =0.0089

Therefore the correct option is 2.

Learn more about probability here:https://brainly.com/question/15308025

#SPJ1

     

Other Questions
What is the mean of the given set of data 12 20 8 15 16 14 7 5 11 8 14? Does QuickBooks Online Plus provides project profitability tracking? which of the following sentences is correct as written? [hint--study correct semicolon usage]. after john ran with the bulls in pamplona; he no longer craved adventure. the great white shark attacked the boy; however, when it ate his hands and feet. russell westbrook averaged a triple double in the 2018 nba season; as a result, he beat oscar robertson's record from the early 1960s. lamar jackson is a dual-threat nfl quarterback; who can use his cannon-like arm to hit receivers in stride, or outrun linebackers with his speed and athleticism. what is the context switch time, associated with swapping, if a disk drive with a transfer rate of 2 mb/s is used to swap out part of a program that is 200 kb in size? assume that no seeks are necessary and that the average latency is 15 ms. the time should reflect only the amount of time necessary to swap out the process. what are the predicted sales if price of clorox is $2, the price of purex is also $2, the price of hcf is $1.5, and hcf is not on display? What foods should be kept separate to avoid contamination? Which theme are found in The Odyssey? Hiram is a computer engineer and, while unemployed, invents a switching device for computer networks. He patents the device, but does not reduce it to practice. Hiram has a zero tax basis for the patent. In consideration of $800,000 plus a $1 royalty per device sold, Hiram assigns the patent to a computer manufacturing company. Hiram assigned all substantial rights in the patent. Which of the following is correct? Is Gizmo the good gremlin? What is a preliminary election held to select candidates or delegates to party conventions? Which of the following describes the independent variable? It is called the respondent variable. It is plotted on the vertical axis when graphed. It refers to a quantity changed by the experimenter. It is something that remains controlled during trials. What does Squealer admit about Napoleon and the windmill Why do the animals accept this? What benefits for the EU countries of the single market? What are two distinguishing characteristics of haikus? Explain the importance of the gardens briefly in promoting public health and cleanliness during the Mughal rule. Which SDG goals can you link to it? Which of the following arguments drew large support for passing the twenty-sixth amendment, granting eighteen-year-olds the right to vote? wilderness area in Belarus and PolandBialowieza Forestautobahnenglobal warmingChunnel"black triangle" returns: logits: tensor of shape (n,num classes,h,w) representing class scores at each pixel yhat: tensor of shape (n,h,w) representing predicted labels at each pixel main loss: loss computed on output of final classifier aux loss:loss computed on output of auxiliary classifier (from intermediate output). note: aux loss is set to a dummy value, since we are not using an auxiliary classifier here, but we keep the same api as pspnet in the next section Reasons that human populations historically have settled in floodplains? In 2021, David, age 66, had adjusted gross income of $32,000. During the year he paid the following medical expenses Prescription medicines Doctors Medical care insurance Over-the-counter hair growth tonic $200 $2,500 $1,400 $250 What amount can David deduct as medical expenses (after the adjusted gross income limitation) in calculating his itemized deductions for 2021? O a. $1,700 O b. None of these is correct O c. $4,100 Od.s0 O e. $900